Đến nội dung

Hình ảnh

Chứng minh rằng $\frac{x^{3}}{y^{3}}+\frac{y^{3}}{z^{3}}+\frac{z^{3}}{x^{3}}$


  • Please log in to reply
Chủ đề này có 9 trả lời

#1
shinichikudo201

shinichikudo201

    Thiếu úy

  • Thành viên
  • 521 Bài viết

Mình có bài này mong các bạn giải giùm:

Cho x; y; z>0. Chứng minh rằng:

$\frac{x^{3}}{y^{3}}+\frac{y^{3}}{z^{3}}+\frac{z^{3}}{x^{3}}\geq \frac{x^{2}}{y^{2}}+\frac{y^{2}}{z^{2}}+\frac{z^{2}}{x^{2}}$

Mình đang học lớp 8.

Thanks.

 


It is the quality of one's convictions that determines successnot the number of followers


#2
nghiemthanhbach

nghiemthanhbach

    $\sqrt{MF}'s\;friend$

  • Thành viên
  • 1056 Bài viết

đặt ẩn a,b,c lần lượt cho từng phân số trên ( không đế luỹ thừa nhé)

$\rightarrow a^3+b^3+c^3\geq a^2+b^2+c^2$

đây trở thành một bài toán cơ bản rồi :)



#3
shinichikudo201

shinichikudo201

    Thiếu úy

  • Thành viên
  • 521 Bài viết

Sao không để phân số giải cho nhanh


It is the quality of one's convictions that determines successnot the number of followers


#4
Trang Luong

Trang Luong

    Đại úy

  • Thành viên
  • 1834 Bài viết

đặt ẩn a,b,c lần lượt cho từng phân số trên ( không đế luỹ thừa nhé)

$\rightarrow a^3+b^3+c^3\geq a^2+b^2+c^2$

đây trở thành một bài toán cơ bản rồi :)

Bạn có thể chứng minh đc BĐT này k $\rightarrow a^3+b^3+c^3\geq a^2+b^2+c^2$  :icon6:  :icon6: 


"Nếu bạn hỏi một người giỏi trượt băng làm sao để thành công, anh ta sẽ nói với bạn: ngã, đứng dậy là thành công"
Issac Newton

#5
nghiemthanhbach

nghiemthanhbach

    $\sqrt{MF}'s\;friend$

  • Thành viên
  • 1056 Bài viết

quên có cách 3 ko bạn

share luôn đi

:))



#6
nguyentrungphuc26041999

nguyentrungphuc26041999

    Sĩ quan

  • Thành viên
  • 406 Bài viết

Mình có bài này mong các bạn giải giùm:

Cho x; y; z>0. Chứng minh rằng:

$\frac{x^{3}}{y^{3}}+\frac{y^{3}}{z^{3}}+\frac{z^{3}}{x^{3}}\geq \frac{x^{2}}{y^{2}}+\frac{y^{2}}{z^{2}}+\frac{z^{2}}{x^{2}}$

Mình đang học lớp 8.

Thanks.

 

Bạn có thể chứng minh đc BĐT này k $\rightarrow a^3+b^3+c^3\geq a^2+b^2+c^2$  :icon6:  :icon6: 

ta chứng minh

$a^{3}+b^{3}+c^{3}\geq a^{2}+b^{2}+c^{2}$

ta có $a^{3}+a^{3}+1\geq 3a^{2}$

$b^{3}+b^{3}+1\geq b^{2}$

$c^{3}+c^{3}+1\geq c^{2}$

cộng lại ta có

$2\left ( a^{3}+b^{3}+c^{3}\right )\geq 3\left ( a^{2}+b^{2}+c^{2} \right )-3$

bây giờ ta cần chứng minh $a^{2}+b^{2}+c^{2}\geq 3$

hiển nhiên đúng vì $a^{2}+b^{2}+c^{2}=\frac{x^{2}}{y^{2}}+\frac{y^{2}}{z^{2}}+\frac{x^{2}}{y^{2}}\geq 3$

dấu bằng xảy ra khi x=y=z



#7
nghiemthanhbach

nghiemthanhbach

    $\sqrt{MF}'s\;friend$

  • Thành viên
  • 1056 Bài viết

Sao không để phân số giải cho nhanh

lý do là giải ra thì nó lằng nhằng ngứa mắt (chủ yếu là nhìn mệt), đặt ra chứng minh vừa dễ vừa đẹp phải không?

:)



#8
kfcchicken98

kfcchicken98

    Thượng sĩ

  • Thành viên
  • 259 Bài viết

Mình có bài này mong các bạn giải giùm:

Cho x; y; z>0. Chứng minh rằng:

$\frac{x^{3}}{y^{3}}+\frac{y^{3}}{z^{3}}+\frac{z^{3}}{x^{3}}\geq \frac{x^{2}}{y^{2}}+\frac{y^{2}}{z^{2}}+\frac{z^{2}}{x^{2}}$

Mình đang học lớp 8.

Thanks.

Bài này có thể giải như sau
$\frac{x^{3}}{y^{3}} + \frac{x^{3}}{y^{3}} + 1 \geq 3.\frac{x^{2}}{y^{2}}$$\frac{y^{3}}{z^{3}} + \frac{y^{3}}{z^{3}} + 1 \geq 3. \frac{y^{2}}{z^{2}}$$\frac{z^{3}}{x^{3}} + \frac{z^{3}}{x^{3}} + 1 \geq 3.\frac{z^{2}}{x^{2}}$
Nên có $2. \left ( \frac{x^{^{3}}}{y^{3}} + \frac{y^{3}}{z^{3}} + \frac{z^{3}}{x^{3}}\right ) \geq 3.\left ( \frac{x^{2}}{y^{2}} + \frac{y^{2}}{z^{2}} + \frac{z^{2}}{x^{2}}\right ) - 3$
có $3\leq \frac{x^{3}}{y^{3}} + \frac{y^{3}}{z^{3}} + \frac{z^{3}}{x^{3}}$
nên sẽ có $3. \left ( \frac{x^{3}}{y^{3}} + \frac{y^{3}}{z^{3}} + \frac{z^{3}}{x^{3}} \right ) \geq 3.\left ( \frac{x^{2}}{y^{2}} + \frac{y^{2}}{z^{2}} + \frac{z^{2}}{x^{2}}\right )$, từ đó suy ra đpcm


Bài viết đã được chỉnh sửa nội dung bởi kfcchicken98: 14-09-2013 - 02:28


#9
shinichikudo201

shinichikudo201

    Thiếu úy

  • Thành viên
  • 521 Bài viết

Bài này có thể giải như sau
$\frac{x^{3}}{y^{3}} + \frac{x^{3}}{y^{3}} + 1 \geq 3.\frac{x^{2}}{y^{2}}$$\frac{y^{3}}{z^{3}} + \frac{y^{3}}{z^{3}} + 1 \geq 3. \frac{y^{2}}{z^{2}}$$\frac{z^{3}}{x^{3}} + \frac{z^{3}}{x^{3}} + 1 \geq 3.\frac{z^{2}}{x^{2}}$
Nên có $2. \left ( \frac{x^{^{3}}}{y^{3}} + \frac{y^{3}}{z^{3}} + \frac{z^{3}}{x^{3}}\right ) \geq 3.\left ( \frac{x^{2}}{y^{2}} + \frac{y^{2}}{z^{2}} + \frac{z^{2}}{x^{2}}\right ) - 3$
có $3\leq \frac{x^{3}}{y^{3}} + \frac{y^{3}}{z^{3}} + \frac{z^{3}}{x^{3}}$
nên sẽ có $3. \left ( \frac{x^{3}}{y^{3}} + \frac{y^{3}}{z^{3}} + \frac{z^{3}}{x^{3}} \right ) \geq 3.\left ( \frac{x^{2}}{y^{2}} + \frac{y^{2}}{z^{2}} + \frac{z^{2}}{x^{2}}\right )$, từ đó suy ra đpcm

Có vẻ bạn làm hơi dài thì phải


It is the quality of one's convictions that determines successnot the number of followers


#10
Thao Hien

Thao Hien

    Hạ sĩ

  • Thành viên
  • 66 Bài viết

$\frac{a^{3}}{b^{3}}+1=\frac{a^{3}+b^{3}}{b^{3}}\geq \frac{ab(a+b)}{b6{3}}=\frac{a^{2}}{b^{2}}+\frac{a}{b}$

$cmtt :... =>VT+3\geq \sum \frac{a^{2}}{b^{2}}+3\sqrt[3]{\frac{a}{b}.\frac{b}{c}.\frac{c}{a}}=VP+3 => VT\geq VP$






1 người đang xem chủ đề

0 thành viên, 1 khách, 0 thành viên ẩn danh